Exercice sur Suite ! (série des inverses)


  • J

    Bonsoir !
    Voila , un petit exercice que je n'arrive pas a resoudre .

    données:
    un=u_{n}=un=$${p=1}$^n$ 1/p
    w</em>n=w</em>{n}=w</em>n=$$_{p=n+1}$^{2n}$ 1/p

    1. Demontrer que , pour tout entier naturel non nul n , Wn ≥ 1/2 .

    --> donc , j'utilise la reccurecne , et je trouve : Wn+1W_{n+1}Wn+1 = Wn + 1/[2(n+1)(2n+1)] , et comme n≥1 , alors Wn+1W_{n+1}Wn+1≥ 1/2 , puis je conclus .

    1. Etablir , pour tout entier naturel non nul m , l'egalité U2m+1U_{2m+1}U2m+1 - U2mU_{2m}U2m = W2mW_{2m}W2m

    Je m'embrouille avec pleins de calcules qui sont faux...

    1. En deduire que (Un) n'est pas majorée .
    2. Conclure sur la convergence du (Un)

    Voila merci beaucoup si vous pouvez bien m'aider !
    A+


  • N

    Salut!

    Exercice sur les suites... hum, c'est plutôt des séries! ...enfin bref!

    Bon, le 1er, par récurrence, tu dois t'en sortir (dis quand même si t'as des soucis) 😉

    Pour le 2 :

    U2n+1U_{2n+1}U2n+1 = ∑$${p=(2n+1)+1}$^{2(2n+1)}$ (1/p)
    soit U</em>2n+1U</em>{2n+1}U</em>2n+1 = ∑ $${p=2n+2}$^{4n+2}$ (1/p)
    Pour U</em>2nU</em>{2n}U</em>2n tu fais pareil!

    Ensuite, pour faire la différence, il faudra que tu fasses un "changement d'indice"! Est-ce que tu sais comment faire? 😕

    %%%
    Prenons un exemple simple : (autre que le tien 😁 )
    Posons UnU_nUn = ∑ $${k=0}$^n$ k
    alors U</em>n+1U</em>{n+1}U</em>n+1 = ∑ $$_{k=0}$^{n+1}$ k

    Si on fait Un+1U_{n+1}Un+1 - UnU_nUn = ∑ $${k=0}$^{n+1}$ k - ∑ $${k=0}$^n$ k
    La fonction se trouvant sous les 2 signes "∑", tu peux faire un changement d'indice :
    ie tu peux écrire :
    Un+1U_{n+1}Un+1 = ∑ $${k=0}$^{n+1}$ k = 1 + 2+ ... + n + (n+1) = ∑ $${k=0}$^n$ k + (n+1)

    Vois-tu comment j'ai obtenu cela? 😲 comme c'est une somme, je peux sortir des éléments du signe ∑ pour les rajouter ensuite! [ tu peux aussi rajouter un terme pour le retrancher ensuite! Style : Un+1U_{n+1}Un+1 = ∑ $${k=0}$^{n+1}$ k = 1 + 2+ ... + n + (n+1) = ∑ $${k=0}$^{n+2}$ k - (n+2) ]

    De là, mon exemple est ensuite tout bête :
    Un+1U_{n+1}Un+1 - UnU_nUn = ∑ $${k=1}$^n$ k + (n+1) - ∑ $${k=1}$^n$ k

    Oh et comme par hasard les 2 "∑" se compensent et il ne reste plus que :
    Un+1U_{n+1}Un+1 - UnU_nUn = (n+1)

    Ensuite tu peux exploiter ce résultat : tu étudies la limite quand n -> +∞ , ça tend toujours vers +∞ donc Un+1U_{n+1}Un+1 > UnU_nUn donc elle est croissante!
    ... enfin, voilà! Tu vois comment il faut faire?
    %%%

    Revenons à ton exercice :

    le 3 : (Un(U_n(Un) n'est pas majorée (c'est à dire qu'elle est toujours inférieure à une certaine valeur) : le mieux serait que tu encadres UnU_nUn (si tu n'y arrives pas, moi ou quelqu'un d'autre t'aidera), et en étudiant la limite des fonctions qui encadrent UnU_nUn, tu devrais trouver ±∞.
    Là : d'après le théorème d'encadrement (plus connu sous le nom de théorème des gendarmes!!! 😆 ), tu pourras conclure que lim $_{n -> +∞}$ UnU_nUn = ±∞ donc (Un(U_n(Un) n'est pas majorée, et on peut même dire qu'elle diverge!!! (ah ben.... c'est ta question4 !)

    Voilà!
    J'espère que je ne t'ai pas trop perturbé avec mes explications longues 😲 et pas toujours claires :rolling_eyes: ! Si tu as un problème, n'hésites surtout pas à revenir... j'essayerai d'être plus claire!

    Bisous Bisous!!! :razz:


  • J

    Le 1 j'ai deja repondu .
    Alors , pour le 2. c'est m et non n , mais sa importe peu la 😄

    Sinon , je sais quand meme faire , mais c'est au niveau de l'egalité de Um avec Wm que je ne comprenais pas :

    U2(m+1)U_{2(m+1)}U2(m+1) - U2mU_{2m}U2m = ∑p=12m+21/p−∑p=12m1/p=∑p=12m1/p−∑p=12m1/p+1(2m+1)+1(2m+2)=2m+2(2m+1)(2m+2)+2m+1(2m+2)(2m+1)=4m+34m2+6m+2\displaystyle \sum_{p=1}^{2m+2} 1/p- \displaystyle \sum_{p=1}^{2m} 1/p= \sum_{p=1}^{2m} 1/p - \sum_{p=1}^{2m} 1/p + \frac{1}{(2m+1)}+\frac{1}{(2m+2)}= \frac{2m+2}{(2m+1)(2m+2)}+\frac{2m+1}{(2m+2)(2m+1)}=\frac{4m+3}{4m^2+6m+2}p=12m+21/pp=12m1/p=p=12m1/pp=12m1/p+(2m+1)1+(2m+2)1=(2m+1)(2m+2)2m+2+(2m+2)(2m+1)2m+1=4m2+6m+24m+3

    Et :

    W2mW_{2m}W2m=∑p=2m+14m1/p=12m+1+12m+2+...+14m\displaystyle \sum_{p=2m+1}^{4m} 1/p = \frac{1}{2m+1}+\frac{1}{2m+2}+...+\frac{1}{4m}p=2m+14m1/p=2m+11+2m+21+...+4m1

    Voila et la je ne vois pas comment W2mW_{2m}W2m = U2m+1U_{2m+1}U2m+1 - U2mU_{2m}U2m ! Car pour W2m ya deja =12m+1+12m+2+..= \frac{1}{2m+1}+\frac{1}{2m+2}+..=2m+11+2m+21+..

    Voila, pour ensuite peut etre faire la suite ! 😆

    merci a+


  • J

    svp aidez moi !! je voudrais comprendre avoir le controle !
    merci beaucoup


  • Zauctore

    Salut. Il y a un problème.

    D'une part :

    u2m+1−u2m=∑p=12m+11p−∑p=12m1p=12m+1 ;u_{2m+1} - u_{2m} = \sum_{p=1}^{2m+1} \frac1p - \sum_{p=1}^{2m} \frac1p = \frac1{2m+1} \ ;u2m+1u2m=p=12m+1p1p=12mp1=2m+11 ;

    D'autre part :

    w2m=∑p=2m+14m1pw_{2m} = \sum_{p=2m+1}^{4m} \frac1pw2m=p=2m+14mp1

    Ces deux quantités ont peu de chance d'être égales ... erreur d'énoncé ?

    Par contre, on a

    u2m−um=wmu_{2m} - u_m = w_{m}u2mum=wm

    puisque u2m−um=∑p=12m1p−∑p=1m1p=12m+⋯+1m+2+1m+1.u_{2m} - u_{m} = \sum_{p=1}^{2m} \frac1p - \sum_{p=1}^{m} \frac1p = \frac1{2m} + \cdots +\frac1{m+2} + \frac1{m+1}.u2mum=p=12mp1p=1mp1=2m1++m+21+m+11.


  • J

    oui erreur denoncé , c'est bon ! c'etait en puissance les m : 2^m .. !
    merci beaucoup !


  • Zauctore

    En puissance ?

    Tu veux dire comme ça : u2m+1−u2mu_{2^{m+1}} - u_{2^m}u2m+1u2m et w2mw_{2^m}w2m ?


  • J

    voila !
    Et donc , je doit avoir une erreur de calcule car le resultat est encore faux ..

    un=∑p=1n1/pun = \displaystyle \sum_{p=1}^{n} 1/pun=p=1n1/p

    wn=∑p=n+12n1/pwn = \displaystyle \sum_{p=n+1}^{2n} 1/pwn=p=n+12n1/p

    Donc pour U2mU_{2m}U2m :
    Ma question est : est-ce que le n=mn=^mn=m , ou le n=2mn=2^mn=2m ?

    Pour n=mn=^mn=m :

    un=∑p=1m1/pu_{n} = \displaystyle \sum_{p=1}^{m} 1/pun=p=1m1/p

    un+1=∑p=1m+11/pu_{n+1} = \displaystyle \sum_{p=1}^{m+1} 1/pun+1=p=1m+11/p

    wn=∑p=m+12m1/pwn = \displaystyle \sum_{p=m+1}^{2m} 1/pwn=p=m+12m1/p

    ou pour n=2mn=2^mn=2m :

    un=∑p=12m1/pu_{n} = \displaystyle \sum_{p=1}^{2^m} 1/pun=p=12m1/p

    un+1=∑p=12m+11/pu_{n+1} = \displaystyle \sum_{p=1}^{2^{m+1}} 1/pun+1=p=12m+11/p

    wn=∑p=2m+12∗2m1/pw_{n} = \displaystyle \sum_{p=2^m + 1}^{2*2^m} 1/pwn=p=2m+122m1/p

    Alors , si n=2mn=2^mn=2m:

    $u_{n+1} - u_{n} =\displaystyle \sum_{p=1}^{2^{m}+1} 1/p - \displaystyle \sum_{p=1}^{2^m} 1/p =\frac {1}{2^{m} + 1$

    Et wn=∑p=2m+12∗2m1/p=12m+1+12m+2+...+12m+1w_{n} = \displaystyle \sum_{p=2^m + 1}^{2*2^m} 1/p = \frac {1}{2^m + 1 } + \frac {1}{2^m + 2 } + ... + \frac {1}{2^{m+1} }wn=p=2m+122m1/p=2m+11+2m+21+...+2m+11

    Donc Wn≠ Un+1U_{n+1}Un+1 - UnU_nUn , pour n = 2m2^m2m

    Maintenant pour n=mn=^mn=m :

    un+1−un=∑p=1m+11/p−∑p=1m1/p=1m+1u_{n+1} - u_{n} =\displaystyle \sum_{p=1}^{m+1} 1/p - \displaystyle \sum_{p=1}^{m} 1/p =\frac {1}{m+1}un+1un=p=1m+11/pp=1m1/p=m+11

    Et wn=∑p=m+122m1/p=1m+1+1m+2+...+122mw_{n} = \displaystyle \sum_{p=m+1}^{2^{2m}} 1/p = \frac {1}{m + 1 } + \frac {1}{m + 2 } + ... + \frac {1}{2^{2m} }wn=p=m+122m1/p=m+11+m+21+...+22m1

    Donc Wn≠ Un+1U_{n+1}Un+1 - UnU_nUn , pour n =m=^m=m

    Donc , ou est l'erreur ?

    Merci !
    a+


  • Zauctore

    L'erreur est dans ceci, pour n=2m+1n=2^{m+1}n=2m+1, tu as : u2m+1=∑p=12m+11/pu_{2^{m+1}} = \sum_{p=1}^{2^{m+1}}1/pu2m+1=p=12m+11/p qui est différent de ∑p=12m+11/p\sum_{p=1}^{2^{m}+1} 1/pp=12m+11/p.

    Félicitations pour tes efforts de LaTeX !

    Remarque: le code pour l'expression de u2m+1u_{2^{m+1}}u2m+1 est
    U_{2^{m+1}} = \sum_{p=1}^{2^{m+1}} 1/p.


  • J

    donc :

    wn=∑p=n+12n1/pwn = \displaystyle \sum_{p=n+1}^{2n} 1/pwn=p=n+12n1/p d'accord ?

    Donc w2m=∑p=(2m)+12∗2m1/pw_{2^m} = \displaystyle \sum_{p=(2^m) +1}^{2*2^m} 1/pw2m=p=(2m)+122m1/p
    Donc w2m=1(2m)+1+1(2m)+2+..+12m+1w_{2^m} = \frac {1}{(2^m) + 1 } + \frac {1}{(2^m) + 2 } +..+ \frac {1}{2^{m+1}}w2m=(2m)+11+(2m)+21+..+2m+11

    Et U(n+1)-Un = $\frac {1}{2^{m+1}$

    Donc c'est bien different entre U2m+1U2^{m+1}U2m+1- U2mU2^mU2m et W2mW2^mW2m

    Voila ?

    Merci a toi en tout cas de bien vouloir m'aider !


  • J

    c'est bon je connais mon erreur ! c'est sur Un+1 !


  • J

    w2m=1(2m)+1+1(2m)+2+..+12m+1w_{2^m} = \frac {1}{(2^m) + 1 } + \frac {1}{(2^m) + 2 } +..+ \frac {1}{2^{m+1}}w2m=(2m)+11+(2m)+21+..+2m+11

    Et U(U(U({2m+1})−U</em>2m)-U</em>{2m})U</em>2m = 12m+1+12m+2+..+12m+1\frac {1}{2^m +1 } + \frac {1}{2^m +2} + .. + \frac {1}{2^{m+1} }2m+11+2m+21+..+2m+11

    Voila , donc W2mW_{2m}W2m = UU(+1)U_{U(+1)}UU(+1) - Un .

    Donc , apres avoir fais cela , en deduire :
    b. Deduire que :

    w2m+1=1+∑k=1mw2kw_{2^{m+1} }= 1 + \displaystyle \sum_{k=1}^{m} w_{2^k}w2m+1=1+k=1mw2k

    Donc :

    w2k=∑p=2k+12k+1=12k+1+12k+2+..+12k+1w_{2^k}= \displaystyle \sum_{p=2^k +1}^{2^{k+1}} = \frac {1}{2^k +1} + \frac {1}{2^k +2} + .. + \frac {1}{2^{k+1}}w2k=p=2k+12k+1=2k+11+2k+21+..+2k+11

    Donc :

    ∑k=1mw2k=(121+1+121+2+..+121+1)+(122+1+122+2+..+122+1)+(123+1+123+2+..+123+1)+..+(12m+1+12m+2+..+12m+1)\displaystyle \sum_{k=1}^{m} w_{2^k}= ( \frac {1}{2^1+1} + \frac {1}{2^1+2} + .. + \frac {1}{2^{1+1}}) + ( \frac {1}{2^2+1} + \frac {1}{2^2+2} + .. + \frac {1}{2^{2+1}}) + ( \frac {1}{2^3+1} + \frac {1}{2^3+2} + .. + \frac {1}{2^{3+1}}) + .. + (\frac {1}{2^m+1} + \frac {1}{2^m+2} + .. + \frac {1}{2^{m+1}})k=1mw2k=(21+11+21+21+..+21+11)+(22+11+22+21+..+22+11)+(23+11+23+21+..+23+11)+..+(2m+11+2m+21+..+2m+11)

    =(13+14)+(15+16+17+18)+(19+110)+(111+112+...+116)+(12m+1+12m+2+..+12m+1)=13+14+15+...+12m+1)= ( \frac {1}{3} + \frac {1}{4}) + ( \frac {1}{5} + \frac {1}{6} + \frac {1}{7} + \frac {1}{8})+ (\frac {1}{9} + \frac {1}{10}) + ( \frac {1}{11} + \frac {1}{12} + ... + \frac {1}{16} ) + (\frac {1}{2^m+1} + \frac {1}{2^m+2} + .. + \frac {1}{2^{m+1}}) = \frac {1}{3} + \frac {1}{4} + \frac {1}{5} +... + \frac {1}{2^{m+1}})=(31+41)+(51+61+71+81)+(91+101)+(111+121+...+161)+(2m+11+2m+21+..+2m+11)=31+41+51+...+2m+11)

    Donc :
    1+∑k=1mw2k=1+13+14+15+...+12m+11 + \displaystyle \sum_{k=1}^{m} w_{2^k} = 1 + \frac {1}{3} + \frac {1}{4} + \frac {1}{5} +... + \frac {1}{2^{m+1}}1+k=1mw2k=1+31+41+51+...+2m+11

    Et u2m+1=1+12+13+14+15+...+12m+1u_{2^{m+1}} = 1 + \frac {1}{2} + \frac {1}{3} + \frac {1}{4} + \frac {1}{5} +... + \frac {1}{2^{m+1}}u2m+1=1+21+31+41+51+...+2m+11

    Donc , je remarque qu'il manque un '1/2' , dans 1+∑k=1mw2k1 + \displaystyle \sum_{k=1}^{m} w_{2^k}1+k=1mw2k , ou est mon erreur ? ! je ne la voi pas .

    Merci beaucoup !

    ps : le plus dure dans l'exercice , c'est de l'ecrire avec Latex 🙂


  • J

    cela doit etre une erreur d'enoncer !


  • Zauctore

    Toi t'es du genre à aimer le labeur... quel effort de code ! sans compter les calculs.

    Sers-toi plutôt de $w_{2^k} = u_{2^{k+1} - u_{2^k}$ pour alléger les calculs, car alors la somme entre 1 et m des w2kw_{2^k}w2k soit sacrément s'arranger par "télescopage".

    Le problème du 1/2 vient de ce que ta série des w2kw_{2^k}w2k a ses indices entre 1 et m ; s'ils étaient entre 0 et m, ça irait mieux...

    c'est sans doute une erreur d'énoncé (et pas d'énoncer).

    @+


  • J

    oki , merci , oui il faudrait que ça commence pour k=0 , pour avoir ce 1/2 !
    désolée pour l'orthographe .

    a+ merci pour tout !


Se connecter pour répondre